Which one of the following pairs of employees is such that at least one member of the pair volunteers?

MayaM on August 3, 2022

Why not A?

In the chain that we create with the conditionals, we see that not L invokes T so why doesn't A work?

Replies
Create a free account to read and take part in forum discussions.

Already have an account? log in

MayaM on August 3, 2022

Sorry - not L invokes T which in turn invokes not F. Confused why A wouldn't work!

Emil-Kunkin on August 6, 2022

Hi MayaM,

This question is asking us which of the answer choices must always have at least one in. That is, the correct answer is one in which we cannot have both of them out. We can have both not F and not T. IF we have

Y:
N: F, T

We also know that M is out, and that R is out, so then L is in, so

Y: L
N: F, T, R, M

This seems like a valid scenario, so it is not A.

MayaM on August 9, 2022

Thank you!